Login

Welcome, Guest. Please login or register.

March 29, 2024, 12:51:45 am

Author Topic: VCE Physics Question Thread!  (Read 603235 times)  Share 

0 Members and 9 Guests are viewing this topic.

Jaswinder

  • Victorian
  • Trendsetter
  • **
  • Posts: 152
  • Respect: 0
  • School Grad Year: 2014
Re: VCE Physics Question Thread!
« Reply #255 on: October 19, 2013, 11:49:31 am »
0
The answer is A, but shouldnt it be the other way around? SO negative current at start then positive? (because of the EMF being (-) change in flux?) Thanks

bonappler

  • Victorian
  • Forum Regular
  • **
  • Posts: 58
  • Respect: +3
Re: VCE Physics Question Thread!
« Reply #256 on: October 19, 2013, 12:01:31 pm »
+2
The answer is A, but shouldnt it be the other way around? SO negative current at start then positive? (because of the EMF being (-) change in flux?) Thanks
I don't think it matters, remember the motion has to be relative.
How do you do this question?
Spoiler

What happens between the two?
Remember when there are two South poles to draw a dotted line (similar to an asymptote in maths) between the two. Then draw your field lines going from the two south poles and behaving asymptotically with the line you have drawn. I am pretty sure you can also draw some conventional lines from the north pole to the south pole as well. It's pretty hard to describe so I am sorry if you do not understand. Maybe someone can upload a picture.

lolipopper

  • Victorian
  • Forum Obsessive
  • ***
  • Posts: 317
  • I'm making aaaalll kaaindzzz of gaains
  • Respect: -4
  • School: Lalor North Secondary College
  • School Grad Year: 2013
Re: VCE Physics Question Thread!
« Reply #257 on: October 19, 2013, 12:26:38 pm »
+1
How do you do this question?
Spoiler

What happens between the two?

its explained in the pic
2014: Monash University, Law

barydos

  • Victorian
  • Forum Obsessive
  • ***
  • Posts: 314
  • Respect: 0
  • School: Nossal High School
  • School Grad Year: 2013
Re: VCE Physics Question Thread!
« Reply #258 on: October 19, 2013, 01:17:15 pm »
0
I don't think it matters, remember the motion has to be relative.Remember when there are two South poles to draw a dotted line (similar to an asymptote in maths) between the two. Then draw your field lines going from the two south poles and behaving asymptotically with the line you have drawn. I am pretty sure you can also draw some conventional lines from the north pole to the south pole as well. It's pretty hard to describe so I am sorry if you do not understand. Maybe someone can upload a picture.
its explained in the pic

Thanks both of you :D

Another question haha
How do you draw the graph of question 11: http://www.vcaa.vic.edu.au/Documents/exams/physics/2011physics2-w.pdf
I thought it would've contained some "rectangular graphs" because constant change in flux = constant voltage? but apparently the graph is a curve.
This leaves me wondering why the answer to electric power q8: http://www.vcaa.vic.edu.au/Documents/exams/physics/2008physics2-w.pdf  is option C.
I would have thought they'd produce the same graphs (looking like q8 option C)

Help please
2012: Methods [47] | Chinese SL [35]
2013: Specialist [48] | Chemistry [41] | Physics [44] | English Language [40]
ATAR: 99.55

SocialRhubarb

  • Victorian
  • Forum Obsessive
  • ***
  • Posts: 313
  • Respect: +34
  • School Grad Year: 2013
Re: VCE Physics Question Thread!
« Reply #259 on: October 19, 2013, 03:22:08 pm »
+3
The difference in the questions is that the question in the 2008 VCAA exam states that the field can be considered 0 outside the poles, while it cannot be assumed so in the 2011 paper. In terms of realism, the curved graph is a more accurate representation, but sometimes for simplicity's sake, we make assumptions like no field outside the poles.
Fight me.

barydos

  • Victorian
  • Forum Obsessive
  • ***
  • Posts: 314
  • Respect: 0
  • School: Nossal High School
  • School Grad Year: 2013
Re: VCE Physics Question Thread!
« Reply #260 on: October 19, 2013, 09:37:02 pm »
0
The difference in the questions is that the question in the 2008 VCAA exam states that the field can be considered 0 outside the poles, while it cannot be assumed so in the 2011 paper. In terms of realism, the curved graph is a more accurate representation, but sometimes for simplicity's sake, we make assumptions like no field outside the poles.

Didn't even notice that small detail, thanks :)
2012: Methods [47] | Chinese SL [35]
2013: Specialist [48] | Chemistry [41] | Physics [44] | English Language [40]
ATAR: 99.55

SocialRhubarb

  • Victorian
  • Forum Obsessive
  • ***
  • Posts: 313
  • Respect: +34
  • School Grad Year: 2013
Re: VCE Physics Question Thread!
« Reply #261 on: October 19, 2013, 10:44:39 pm »
+1
The net force, which is the centripetal force in this case, goes towards the centre of the circular path of rotation.

If you think about it, if the net force was towards the centre of the chain, the person in the device would accelerate upwards, towards the centre of the chain, but they clearly instead follow a circular path.

We find the magnitude of the tension by realising that the net force must be the centripetal force, and so the vertical component of the tension must negate the person's weight force, while the horizontal component of the tension provides the centripetal force. With these two figures, we can calculate the magnitude of the tension in the chain with Pythagoras' theorem.
Fight me.

lzxnl

  • Victorian
  • ATAR Notes Legend
  • *******
  • Posts: 3432
  • Respect: +215
Re: VCE Physics Question Thread!
« Reply #262 on: October 19, 2013, 11:10:36 pm »
+4
Let's clarify something. The net force is directed radially inwards from the object to the axis of rotation. For a constant axis of rotation, this axis of rotation is perpendicular to the force vector. Think about that. You can see how this works in the diagram.

As for astronauts in outer space, there is no contradiction. The net force is directed inwards in a direction that is perpendicular to the imaginary axis they are revolving around.

You can see that the tension has a horizontal component, which is the net force mv^2/r, and a vertical component to cancel out the weight force. As these components are perpendicular, use Pythag.
2012
Mathematical Methods (50) Chinese SL (45~52)

2013
English Language (50) Chemistry (50) Specialist Mathematics (49~54.9) Physics (49) UMEP Physics (96%) ATAR 99.95

2014-2016: University of Melbourne, Bachelor of Science, Diploma in Mathematical Sciences (Applied Maths)

2017-2018: Master of Science (Applied Mathematics)

2019-2024: PhD, MIT (Applied Mathematics)

Accepting students for VCE tutoring in Maths Methods, Specialist Maths and Physics! (and university maths/physics too) PM for more details

barydos

  • Victorian
  • Forum Obsessive
  • ***
  • Posts: 314
  • Respect: 0
  • School: Nossal High School
  • School Grad Year: 2013
Re: VCE Physics Question Thread!
« Reply #263 on: October 20, 2013, 11:20:31 am »
0
When drawing a circuit, how do we know when the switch has to be opposite a thermistor or LDR rather than a variable resistor (or vice versa)?
2012: Methods [47] | Chinese SL [35]
2013: Specialist [48] | Chemistry [41] | Physics [44] | English Language [40]
ATAR: 99.55

lolipopper

  • Victorian
  • Forum Obsessive
  • ***
  • Posts: 317
  • I'm making aaaalll kaaindzzz of gaains
  • Respect: -4
  • School: Lalor North Secondary College
  • School Grad Year: 2013
Re: VCE Physics Question Thread!
« Reply #264 on: October 20, 2013, 02:19:32 pm »
+1
When drawing a circuit, how do we know when the switch has to be opposite a thermistor or LDR rather than a variable resistor (or vice versa)?

you have to see when the heater must be turned on. If it is to be turned on when the voltage increases then against the element whose resistance increases (Voltage increases across it as well) and vice versa.
« Last Edit: October 20, 2013, 02:26:09 pm by lolipopper »
2014: Monash University, Law

barydos

  • Victorian
  • Forum Obsessive
  • ***
  • Posts: 314
  • Respect: 0
  • School: Nossal High School
  • School Grad Year: 2013
Re: VCE Physics Question Thread!
« Reply #265 on: October 20, 2013, 09:48:28 pm »
0
you have to see when the heater must be turned on. If it is to be turned on when the voltage increases then against the element whose resistance increases (Voltage increases across it as well) and vice versa.

That makes sense, thank you!
2012: Methods [47] | Chinese SL [35]
2013: Specialist [48] | Chemistry [41] | Physics [44] | English Language [40]
ATAR: 99.55

Robert123

  • Victorian
  • Forum Obsessive
  • ***
  • Posts: 201
  • Respect: +5
  • School: Kyabram P-12 College
Re: VCE Physics Question Thread!
« Reply #266 on: October 21, 2013, 08:07:44 pm »
0
This satellite question is driving me crazy!!
Q8 of motion in the 2013 sample exam by VCAA
R=1.35*10^7m
G=6.67*10^-11
M (earth)=5.98*10^24kg
m (satellite)=525kg
What is the period?
Both Vicphysics and Itute answer say it is1.53*10^4s but whenever I do it I always get 1.56*10^4s. It is so close yet so far away, could someone please test this question, I know how to do it but it just isn't working.


random_person

  • Victorian
  • Trendsetter
  • **
  • Posts: 116
  • Respect: +5
Re: VCE Physics Question Thread!
« Reply #267 on: October 21, 2013, 08:57:37 pm »
0
This satellite question is driving me crazy!!
Q8 of motion in the 2013 sample exam by VCAA
R=1.35*10^7m
G=6.67*10^-11
M (earth)=5.98*10^24kg
m (satellite)=525kg
What is the period?
Both Vicphysics and Itute answer say it is1.53*10^4s but whenever I do it I always get 1.56*10^4s. It is so close yet so far away, could someone please test this question, I know how to do it but it just isn't working.

You're right!!!!!!?????????


2012: Further Mathematics [50] (Premier's Award)
2013: English | Physics | Mathematical Methods | Specialist Mathematics | Chemistry
Bachelor of Medicine/Bachelor of Surgery (Honours)

PM me any questions about the UMAT and Monash Medicine!

Thu Thu Train

  • Voted AN's sexiest member 2012
  • Victorian
  • Forum Leader
  • ****
  • Posts: 667
  • <3
  • Respect: +336
Re: VCE Physics Question Thread!
« Reply #268 on: October 21, 2013, 09:57:36 pm »
0
Strange I also get 1.56x104 but two places(who you should assume are correct) are reporting the same answer which implies they've either both made mistakes or we're missing something obvious.
        (
     '( '
    "'  //}
   ( ''"
   _||__ ____ ____ ____
  (o)___)}___}}___}}___}   
  'U'0 0  0 0  0 0  0 0    0 0
BBSN14

i actually almost wish i was a monash student.

Stevensmay

  • Guest
Re: VCE Physics Question Thread!
« Reply #269 on: October 21, 2013, 10:03:53 pm »
0
The answer you have gotten is correct. One upside of physics is that there often aren't very many technicalities to a question, it's just a matter of plugging numbers in.